Which one of the following could be true?

Maria-Marin on May 24, 2020

Correct Answer

Could you please explain why A is wrong and how we get to the correct answer choice?

Reply
Create a free account to read and take part in forum discussions.

Already have an account? log in

Skylar on May 25, 2020

@Maria-Marin, happy to help!

(A) is incorrect because it is impossible for garlic to be added first. This is because Rule #4 tells us that both garlic and onion are in, and Rule #5 tells us that onion must be added before garlic. This means that at least one ingredient must precede garlic, so it cannot be first.

Depending on how you set up this game, you can get to the correct answer through deduced scenarios or through the process of elimination. Let's look at the other incorrect answer choices.

(B) is incorrect because, if garlic was fifth, all seven ingredients would have to be in and that is impossible. This is because Rule #4 says that garlic, kale, onion, and tomato are all in, and Rule #5 says that garlic comes before kale and tomato. So, if garlic is fifth, kale and tomato must fill the sixth and seven spots. However, it is impossible to have all seven spots filled with the seven ingredients because Rule #3 says that either potato or yam must be out.

(C) is incorrect because Rule #1 says that J is either first or last. If it is placed fifth, that means that fifth would have to be the last ingredient added. However this is impossible because Rule #4 says that garlic, kale, onion, and tomato are all added, and Rule #3 says that either potato or yam is also added. This adds up to 5 ingredients before J is added.

(E) is incorrect because it is impossible for tomato to be added second. This is because Rule #4 tells us that galic, kale, onion, and tomato are all in, and Rule #5 tells us that onion must be added before garlic which must be added before tomato. This means that at least two ingredients must precede tomato, so it cannot be second.

This leaves the correct answer- (D). (D) is correct because potato can be placed first without breaking any rules. For example, we could have the sequence: POGKT.

Does that make sense? Please let us know if you have any other questions!